Stay ahead of learning milestones! Enroll in a class over the summer!

G
Topic
First Poster
Last Poster
k a May Highlights and 2025 AoPS Online Class Information
jlacosta   0
May 1, 2025
May is an exciting month! National MATHCOUNTS is the second week of May in Washington D.C. and our Founder, Richard Rusczyk will be presenting a seminar, Preparing Strong Math Students for College and Careers, on May 11th.

Are you interested in working towards MATHCOUNTS and don’t know where to start? We have you covered! If you have taken Prealgebra, then you are ready for MATHCOUNTS/AMC 8 Basics. Already aiming for State or National MATHCOUNTS and harder AMC 8 problems? Then our MATHCOUNTS/AMC 8 Advanced course is for you.

Summer camps are starting next month at the Virtual Campus in math and language arts that are 2 - to 4 - weeks in duration. Spaces are still available - don’t miss your chance to have an enriching summer experience. There are middle and high school competition math camps as well as Math Beasts camps that review key topics coupled with fun explorations covering areas such as graph theory (Math Beasts Camp 6), cryptography (Math Beasts Camp 7-8), and topology (Math Beasts Camp 8-9)!

Be sure to mark your calendars for the following upcoming events:
[list][*]May 9th, 4:30pm PT/7:30pm ET, Casework 2: Overwhelming Evidence — A Text Adventure, a game where participants will work together to navigate the map, solve puzzles, and win! All are welcome.
[*]May 19th, 4:30pm PT/7:30pm ET, What's Next After Beast Academy?, designed for students finishing Beast Academy and ready for Prealgebra 1.
[*]May 20th, 4:00pm PT/7:00pm ET, Mathcamp 2025 Qualifying Quiz Part 1 Math Jam, Problems 1 to 4, join the Canada/USA Mathcamp staff for this exciting Math Jam, where they discuss solutions to Problems 1 to 4 of the 2025 Mathcamp Qualifying Quiz!
[*]May 21st, 4:00pm PT/7:00pm ET, Mathcamp 2025 Qualifying Quiz Part 2 Math Jam, Problems 5 and 6, Canada/USA Mathcamp staff will discuss solutions to Problems 5 and 6 of the 2025 Mathcamp Qualifying Quiz![/list]
Our full course list for upcoming classes is below:
All classes run 7:30pm-8:45pm ET/4:30pm - 5:45pm PT unless otherwise noted.

Introductory: Grades 5-10

Prealgebra 1 Self-Paced

Prealgebra 1
Tuesday, May 13 - Aug 26
Thursday, May 29 - Sep 11
Sunday, Jun 15 - Oct 12
Monday, Jun 30 - Oct 20
Wednesday, Jul 16 - Oct 29

Prealgebra 2 Self-Paced

Prealgebra 2
Wednesday, May 7 - Aug 20
Monday, Jun 2 - Sep 22
Sunday, Jun 29 - Oct 26
Friday, Jul 25 - Nov 21

Introduction to Algebra A Self-Paced

Introduction to Algebra A
Sunday, May 11 - Sep 14 (1:00 - 2:30 pm ET/10:00 - 11:30 am PT)
Wednesday, May 14 - Aug 27
Friday, May 30 - Sep 26
Monday, Jun 2 - Sep 22
Sunday, Jun 15 - Oct 12
Thursday, Jun 26 - Oct 9
Tuesday, Jul 15 - Oct 28

Introduction to Counting & Probability Self-Paced

Introduction to Counting & Probability
Thursday, May 15 - Jul 31
Sunday, Jun 1 - Aug 24
Thursday, Jun 12 - Aug 28
Wednesday, Jul 9 - Sep 24
Sunday, Jul 27 - Oct 19

Introduction to Number Theory
Friday, May 9 - Aug 1
Wednesday, May 21 - Aug 6
Monday, Jun 9 - Aug 25
Sunday, Jun 15 - Sep 14
Tuesday, Jul 15 - Sep 30

Introduction to Algebra B Self-Paced

Introduction to Algebra B
Tuesday, May 6 - Aug 19
Wednesday, Jun 4 - Sep 17
Sunday, Jun 22 - Oct 19
Friday, Jul 18 - Nov 14

Introduction to Geometry
Sunday, May 11 - Nov 9
Tuesday, May 20 - Oct 28
Monday, Jun 16 - Dec 8
Friday, Jun 20 - Jan 9
Sunday, Jun 29 - Jan 11
Monday, Jul 14 - Jan 19

Paradoxes and Infinity
Mon, Tue, Wed, & Thurs, Jul 14 - Jul 16 (meets every day of the week!)

Intermediate: Grades 8-12

Intermediate Algebra
Sunday, Jun 1 - Nov 23
Tuesday, Jun 10 - Nov 18
Wednesday, Jun 25 - Dec 10
Sunday, Jul 13 - Jan 18
Thursday, Jul 24 - Jan 22

Intermediate Counting & Probability
Wednesday, May 21 - Sep 17
Sunday, Jun 22 - Nov 2

Intermediate Number Theory
Sunday, Jun 1 - Aug 24
Wednesday, Jun 18 - Sep 3

Precalculus
Friday, May 16 - Oct 24
Sunday, Jun 1 - Nov 9
Monday, Jun 30 - Dec 8

Advanced: Grades 9-12

Olympiad Geometry
Tuesday, Jun 10 - Aug 26

Calculus
Tuesday, May 27 - Nov 11
Wednesday, Jun 25 - Dec 17

Group Theory
Thursday, Jun 12 - Sep 11

Contest Preparation: Grades 6-12

MATHCOUNTS/AMC 8 Basics
Friday, May 23 - Aug 15
Monday, Jun 2 - Aug 18
Thursday, Jun 12 - Aug 28
Sunday, Jun 22 - Sep 21
Tues & Thurs, Jul 8 - Aug 14 (meets twice a week!)

MATHCOUNTS/AMC 8 Advanced
Sunday, May 11 - Aug 10
Tuesday, May 27 - Aug 12
Wednesday, Jun 11 - Aug 27
Sunday, Jun 22 - Sep 21
Tues & Thurs, Jul 8 - Aug 14 (meets twice a week!)

AMC 10 Problem Series
Friday, May 9 - Aug 1
Sunday, Jun 1 - Aug 24
Thursday, Jun 12 - Aug 28
Tuesday, Jun 17 - Sep 2
Sunday, Jun 22 - Sep 21 (1:00 - 2:30 pm ET/10:00 - 11:30 am PT)
Monday, Jun 23 - Sep 15
Tues & Thurs, Jul 8 - Aug 14 (meets twice a week!)

AMC 10 Final Fives
Sunday, May 11 - Jun 8
Tuesday, May 27 - Jun 17
Monday, Jun 30 - Jul 21

AMC 12 Problem Series
Tuesday, May 27 - Aug 12
Thursday, Jun 12 - Aug 28
Sunday, Jun 22 - Sep 21
Wednesday, Aug 6 - Oct 22

AMC 12 Final Fives
Sunday, May 18 - Jun 15

AIME Problem Series A
Thursday, May 22 - Jul 31

AIME Problem Series B
Sunday, Jun 22 - Sep 21

F=ma Problem Series
Wednesday, Jun 11 - Aug 27

WOOT Programs
Visit the pages linked for full schedule details for each of these programs!


MathWOOT Level 1
MathWOOT Level 2
ChemWOOT
CodeWOOT
PhysicsWOOT

Programming

Introduction to Programming with Python
Thursday, May 22 - Aug 7
Sunday, Jun 15 - Sep 14 (1:00 - 2:30 pm ET/10:00 - 11:30 am PT)
Tuesday, Jun 17 - Sep 2
Monday, Jun 30 - Sep 22

Intermediate Programming with Python
Sunday, Jun 1 - Aug 24
Monday, Jun 30 - Sep 22

USACO Bronze Problem Series
Tuesday, May 13 - Jul 29
Sunday, Jun 22 - Sep 1

Physics

Introduction to Physics
Wednesday, May 21 - Aug 6
Sunday, Jun 15 - Sep 14
Monday, Jun 23 - Sep 15

Physics 1: Mechanics
Thursday, May 22 - Oct 30
Monday, Jun 23 - Dec 15

Relativity
Mon, Tue, Wed & Thurs, Jun 23 - Jun 26 (meets every day of the week!)
0 replies
jlacosta
May 1, 2025
0 replies
Beautiful Angle Sum Property in Hexagon with Incenter
Raufrahim68   0
23 minutes ago
Hello everyone! I discovered an interesting geometric property and would like to share it with the community. I'm curious if this is a known result and whether it can be generalized.

Problem Statement:
Let
A
B
C
D
E
K
ABCDEK be a convex hexagon with an incircle centered at
O
O. Prove that:


A
O
B
+

C
O
D
+

E
O
K
=
180

∠AOB+∠COD+∠EOK=180
0 replies
Raufrahim68
23 minutes ago
0 replies
Anything real in this system must be integer
Assassino9931   7
N 29 minutes ago by Leman_Nabiyeva
Source: Al-Khwarizmi International Junior Olympiad 2025 P1
Determine the largest integer $c$ for which the following statement holds: there exists at least one triple $(x,y,z)$ of integers such that
\begin{align*} x^2 + 4(y + z) = y^2 + 4(z + x) = z^2 + 4(x + y) = c \end{align*}and all triples $(x,y,z)$ of real numbers, satisfying the equations, are such that $x,y,z$ are integers.

Marek Maruin, Slovakia
7 replies
Assassino9931
May 9, 2025
Leman_Nabiyeva
29 minutes ago
CIIM 2011 First day problem 3
Ozc   2
N 39 minutes ago by pi_quadrat_sechstel
Source: CIIM 2011
Let $f(x)$ be a rational function with complex coefficients whose denominator does not have multiple roots. Let $u_0, u_1,... , u_n$ be the complex roots of $f$ and $w_1, w_2,..., w_m$ be the roots of $f'$. Suppose that $u_0$ is a simple root of $f$. Prove that
\[ \sum_{k=1}^m \frac{1}{w_k - u_0} = 2\sum_{k = 1}^n\frac{1}{u_k - u_0}.\]
2 replies
Ozc
Oct 3, 2014
pi_quadrat_sechstel
39 minutes ago
IMO 2009 P2, but in space
Miquel-point   1
N 40 minutes ago by Miquel-point
Source: KoMaL A. 485
Let $ABCD$ be a tetrahedron with circumcenter $O$. Suppose that the points $P, Q$ and $R$ are interior points of the edges $AB, AC$ and $AD$, respectively. Let $K, L, M$ and $N$ be the centroids of the triangles $PQD$, $PRC,$ $QRB$ and $PQR$, respectively. Prove that if the plane $PQR$ is tangent to the sphere $KLMN$ then $OP=OQ=OR.$

1 reply
Miquel-point
40 minutes ago
Miquel-point
40 minutes ago
No more topics!
GCD Functional Equation
pinetree1   61
N Apr 21, 2025 by ihategeo_1969
Source: USA TSTST 2019 Problem 7
Let $f: \mathbb Z\to \{1, 2, \dots, 10^{100}\}$ be a function satisfying
$$\gcd(f(x), f(y)) = \gcd(f(x), x-y)$$for all integers $x$ and $y$. Show that there exist positive integers $m$ and $n$ such that $f(x) = \gcd(m+x, n)$ for all integers $x$.

Ankan Bhattacharya
61 replies
pinetree1
Jun 25, 2019
ihategeo_1969
Apr 21, 2025
GCD Functional Equation
G H J
Source: USA TSTST 2019 Problem 7
The post below has been deleted. Click to close.
This post has been deleted. Click here to see post.
Leo.Euler
577 posts
#57
Y by
Fix any prime $p$ less than $10^{100}$. It suffices to show that the desired holds under $\nu_p$ analysis.

Let $x$ be an integer such that $\nu_p(f(x))$ is maximal. Then for any $y$ distinct from $x$, \[ \nu_p(f(y)) = \nu_p(x - y). \]Taking $n=f(x)$ and $m \equiv -x \pmod{p^{\nu_p(f(x)}}$, $y \neq x$ satisfy the desired. Realize that plugging in $x$ into the desired with these values of $m$ and $n$ returns a valid equality, as desired.
Z K Y
The post below has been deleted. Click to close.
This post has been deleted. Click here to see post.
shendrew7
796 posts
#58
Y by
Suppose $S$ is the set of primes under $10^{100}$, $p$ as an arbitrary prime in $S$, $e_p$ be the maximum possible value $v_p(f(x))$, and $x_p$ be a value of $x$ for which this maximum holds. Using $v_p$ and substituting $(x,y) = (x_p,t)$, we get
\[\min \left(v_p(f(x_p)), v_p(f(t))\right) = \min \left(v_p(f(x_p)), v_p(x_p-t)\right)\]\[\implies v_p(f(t)) = \min(e_p, v_p(x_p-t)).\]
Then we construct our values of $m$ and $n$ such that
\[m = -x_p \pmod{p^{e_p}} \quad \forall \quad  p \in S, \qquad n = \prod_{p \in S} p^{e_p},\]
which works as
\[v_p(\gcd(n, x+m)) = \min(v_p(n), v_p(x+m)) = \min(e_p, v_p(x-x_p)) = v_p(f(x))\]
for all primes $p \in S$ and $x \in \mathbb{Z}$. $\blacksquare$
Z K Y
The post below has been deleted. Click to close.
This post has been deleted. Click here to see post.
Shreyasharma
682 posts
#59
Y by
Ew. Weird, intractable number theory because I'm bad.

The idea is to interpret the condition in $\nu_p$. We are motivated to do so because in terms of $\nu_p$ the desired conclusion can be restated in terms of individual primes.

Then fix a prime $p$. Then over all primes the given can be restated as,
\begin{align*}
\min(\nu_p(f(x)), \nu_p(f(y))) = \min(\nu_p(f(x)), \nu_p(x-y))
\end{align*}Consider $k_p$ such that $\nu_p(f(k_p))$ is maximal. Also let $\nu_p(f(k_p)) = e_p$. Then we find we have,
\begin{align}
\nu_p(f(x)) = \min (e_p, \nu_p(x-k_p))
\end{align}Now a hint told me to use CRT because I'm bad. To finish note that we want to find $m$ and $n$ such that we always have,
\begin{align*}
\nu_p(f(x)) = \min(\nu_p(m+x), \nu_p(n))
\end{align*}over all primes $p$.

Now choose $m$ satisfying,
\begin{align*}
m \equiv -k_p \pmod{p^{e_p}}
\end{align*}over all $p$. A solution is guaranteed by CRT. Similarly for $n$ choose it to satisfy,
\begin{align*}
n \equiv 0 \pmod{p^{e_p}}
\end{align*}for all $p$ which exists, once again, by CRT. Now from $(1)$ our choices of $m$ and $n$ suffice.
Z K Y
The post below has been deleted. Click to close.
This post has been deleted. Click here to see post.
Markas
150 posts
#60
Y by
Take $x_0$ such that $\nu_p(f(x_0))$ is max for some given prime p. Let $x = x_0$, we get $\min(\nu_p(f(x_0)), \nu_p(x_0-y)) = \nu_p(f(y))$. Now take $m \equiv -x_0 \pmod {p^{\nu_p(f(x_0))}}$, $n \equiv f(x_0) \pmod {p^{\nu_p(f(x_0))}}$ $\Rightarrow$ such $f(x)$ satisfies a condition equivalent to the one we wanted. If we use this expression over all valid p, we get a working solution (m, n) by CRT.
Z K Y
The post below has been deleted. Click to close.
This post has been deleted. Click here to see post.
de-Kirschbaum
201 posts
#61
Y by
If we rephrase the problem in terms of $\nu_p$ we have that for all primes $p$, $$\min(\nu_p(f(x)),\nu_p(f(y))=\min(\nu_p(f(x)), \nu_p(x-y))$$for all integers $x,y$. Now since the image of $f$ is bounded, we know that there exists some $c$ such that $\nu_p(f(c))=s$ is maximized. Note that if we fix $x=c$, then that reduces the above condition to $\nu_p(f(y))=\min(s, \nu_p(c-y))$ for every positive integer $y$, so we are done if we can prove that $\min(s, \nu_p(c-y))=\min(\nu_p(m+y),\nu_p(n))$ for some integer $m,n$. Suppose the set of primes contained in $\{1, 2, \ldots, 10^{100}\}$ is $P=\{p_1, \ldots, p_k\}$. Now note that if we take $n=\prod_{p \in P}p_i^{e_{i}}$ where $e_i$ is the maximal power of $p_i$ contained in $\{1, 2, \ldots, 10^{100}\}$, would have $\nu_p(n)$ be maximized for any prime $p$, thus $\nu_p(n)=s$. Now we just need $\nu_p(m+y)=\nu_p(c-y)$ for all $p$. Note that when $c-y \equiv 0 \mod{p_i^{e_i}}$, we have $y \equiv c \mod{p_i^{e_i}}$. That means we just need
\begin{align*}
        m &\equiv -c \mod{p_1^{e_1}}\\
        m &\equiv -c \mod{p_2^{e_2}} \\
        &\vdots
        \\
        m &\equiv -c \mod{p_k^{e_k}}
    \end{align*}By CRT we know such a positive integer $m$ exists. Thus we are done.
Z K Y
The post below has been deleted. Click to close.
This post has been deleted. Click here to see post.
Mathandski
758 posts
#62
Y by
Suspiciously simple solution without modular arithmetic
Attachments:
Z K Y
The post below has been deleted. Click to close.
This post has been deleted. Click here to see post.
numbertheory97
43 posts
#63
Y by
Fun problem!

Solution. For each prime $p < 10^{100}$, let $P_p = p^k$ denote the largest power of $p$ for which $P_p \mid f(x)$ for some $x$. The main idea is the following:

Claim: Suppose $P \mid P_p$. All integers $x$ for which $P \mid f(x)$ are congruent mod $P$.

Proof. Let $x, y$ be integers for which $P \mid f(x), f(y)$, and suppose to the contrary that $\nu_p(x - y) < \nu_p(P) \leq k$. Then \[\min(\nu_p(f(x)), \nu_p(f(y))) \geq k > \nu_p(x - y) = \min(\nu_p(f(x)), \nu_p(x - y))),\]so our assumption was wrong. $\square$

Thus for each $p$, there is a unique integer $a_p \in [0, P_p - 1]$ for which all solutions of $f(x) \equiv 0 \pmod{P_p}$ satisfy $x \equiv a_p \pmod{P_p}$. We now claim that taking $m > 0$ such that $m \equiv -a_p \pmod{P_p}$ for all $p < 10^{100}$ (possible by CRT) and setting $n = \prod_p P_p$ is a valid choice of $(m, n)$.

Indeed, let $p$ be a prime, and $x$ an integer with $\nu_p(f(x)) = \ell$. Since $x \equiv a_p \pmod{p^\ell}$ and $x \not\equiv a_p \pmod{p^{\ell + 1}}$, we have \[\nu_p(\gcd(m + x, n)) = \min(\nu_p(m + x), \nu_p(n)) = \min(\nu_p(x - a_p), \nu_p(n)) = \ell = \nu_p(f(x))\]from the fact that $\nu_p(n) = \nu_p(P_p)$. Combined with the observation that $\nu_p(f(x)) = 0 = \nu_p(n)$ for all primes $p > 10^{100}$, this completes the proof. $\square$

Remark. I had to edit the above proof a bit, since originally I assumed taking $n = \text{lcm}(1, 2, \dots, 10^{100})$ was fine. But after looking at hints, I realized this raises issues when some prime power dividing $\text{lcm}(1, 2, \dots, 10^{100})$ isn't actually in the range of $f$.
Z K Y
The post below has been deleted. Click to close.
This post has been deleted. Click here to see post.
L13832
268 posts
#65
Y by
For a prime $p$ we have $\min(\nu_{p}(f(x)), \nu_{p}(f(y))) = \min(\nu_{p}(f(x), \nu_{p}(x - y))$, let $x$ be the maximal value of $\nu_{p}(f(x))$, so we have $$\nu_{p}(f(k))=\min(\nu_{p}(f(x)),\nu_{p}(k-x))\qquad{(*)}$$We basically want to always have
$\nu_p(f(x)) = \min(\nu_p(m+x), \nu_p(n))$.
Now choose $m\equiv 
-x \pmod{p^{\nu_{p}(f(x))}}$ and $n\equiv f(x)\pmod {p^{\nu_{p}(f(x))}}$ by CRT and we are done!
This post has been edited 1 time. Last edited by L13832, Oct 26, 2024, 5:23 AM
Reason: latex typo and solution error
Z K Y
The post below has been deleted. Click to close.
This post has been deleted. Click here to see post.
pi271828
3371 posts
#66 • 1 Y
Y by megarnie
Define $x_p$ to be the maximal value such that $\nu_p(f(x)) = \ell_p$ and $\ell_p$ be the maximum value it attains. We claim that \begin{align*} n = \prod_{p \le 10^{100}} p^{\ell_p} \\ m \equiv x_p \pmod{p^{\ell_p}} \; \text{for all} \; p \le 10^{100}\end{align*}
work for $(m, n)$. We note that there exists an $m$ satisfying that condition by CRT.

Take a value $L$ and note that $\nu_p(f(a))$ can be found by directly using the given equation. Note that \begin{align*} \operatorname{min}(\nu_p(f(L)), \nu_p(f(x_p))) = \nu_p(f(L)) = \operatorname{min}(\ell_p, \nu_p(L-x_p))\end{align*}Now note that with the given values of $m$ and $n$, we have \begin{align*} \nu_p(f(L)) = \operatorname{min}(\nu_p(L-\operatorname{lcm} \{x_p\}), \nu_p(n)) \\ = \operatorname{min}(\nu_p(L-x_p), \ell_p) \end{align*}so we are done.
Z K Y
The post below has been deleted. Click to close.
This post has been deleted. Click here to see post.
bjump
1028 posts
#67
Y by
Call the primes less than $10^{100}$ in increasing order, $p_1$, $p_2$, $\dots$, $p_{n}$. Where $n$ is the number of primes less than $10^{100}$. Call the smallest positive $x$ such that $\nu_{p_i} (f(x))$ is maximized $x_{p_i}$ for each prime $p_i$. We have
$$\nu_{p_i}(\gcd(f(x_{p_i}), f(x + k \cdot p^{\nu_{p_i} f(x_{p_i})} )) = \nu_{p_i} ( \gcd(f(x_{p_i}), k \cdot p^{\nu_{p_i} f(x_{p_i})} ) ) = \nu_{p_i} ( f(x_{p_i}))$$$$\implies \nu_{p_i} (f(x + k \cdot p^{\nu_{p_i} f(x_{p_i})} )) = \nu_{p_i} (f(x_{p_i}))$$So by CRT there exists a negative value of $x$ call it $x_{\text{ook}}$ such that for each prime $\nu_{p} f(x_{\text{ook}})$ is maximized. Now if $\nu_{p_{i}}(y) = k < \nu_{p_{i}} ( f(x_{\text{ook}}))$ then
$$\nu_{p_i} (\gcd(f(x_{\text{ook}}), f(x_{\text{ook}} + y))) = \nu_{p_i}(\gcd(f(x_{\text{ook}}), y)) = \nu_{p_i}(y)$$So $f(x) = \boxed{ \gcd \left( x - x_{\text{ook}}, \prod_{i=1}^n p_i^{\nu_{p_i} ( f(x_{\text{ook}}))} \right)}$
Z K Y
The post below has been deleted. Click to close.
This post has been deleted. Click here to see post.
D4N13LCarpenter
13 posts
#69 • 1 Y
Y by Vahe_Arsenyan
Here's a nice solution which doesn't look at the $\nu_p$:

Let $x_0$ be such that $f(x_0)$ is the maximum value of $f$ over all possible inputs, which exists as $f$ is upper bounded. We begin by proving the following

Claim 1: $f(x)\mid f(x_0)$ for all $x$
Notice that $f(x)\mid f(x+kf(x))$ $\forall k$ as we have $\gcd(f(x), x-(x+kf(x))=\gcd(f(x), -kf(x))=f(x)$ so $\gcd(f(x),f(x+kf(x))=f(x)$. The key idea now is to prove that there exist $a, b$ such that $x+af(x)=x_0+bf(x_0)$. By rearranging the terms, we obtain $$ af(x)-bf(x_0) = x_0-x$$but we have $\gcd(f(x_0), f(x))\mid x_0-x$ so this is just Bézout's identity.

To finish, note that by the previous observation there exists a $n$ such that $f(x)\mid f(n)$ and $f(x_0)\mid f(n)$. However, we have $f(n)\leq f(x_0)$ by definition, hence $f(n)=f(x_0)$, from which the result is immediate.
We know that $$\gcd(f(x_0), f(y))=\gcd(f(x_0), x_0-y)$$for all $y$, but by Claim 1 $\gcd(f(x_0), f(y))=f(y)$, so this rewrites to $$f(y)=\gcd(f(x_0), x_0-y)$$from where the choices $n=f(x_0)$ and $m=-x_0$ finish the problem.
Z K Y
The post below has been deleted. Click to close.
This post has been deleted. Click here to see post.
mathwiz_1207
100 posts
#70
Y by
Nice problem! We claim that for any positive integer $m$, and $n$ such that
\[v_p(n) = \text{max}\{v_p(x)\}\]over all integers $x$ and primes $p$. $n$ is finite because $v_p(n) = 0$ for all $p > 10^{100}$, and $v_p(n)$ is finite for all $p \leq 10^{100}$. We prove the following claim:


For integers $a, b$ we have
\[\text{min}\{v_p(a-b), v_p(a)\} = \text{min}\{v_p(a), v_p(b)\}\]

Proof. If either $a$ or $b$ is $0$, it is trivial, so assume $a, b$ are nonzero. Then, let $v_p(a) = c$ and $v_p(b) = d$ so $a = a' \cdot p^c$, and $b = b' \cdot p^d$. We take cases.

If $c > d$, the condition is equivalent to
\[\text{min}\{v_p(p^d(a' \cdot p^{c -d} - b')), c\} = \text{min}\{c, d\}\]which is equivalent to $d = d$, true.

If $c = d$, the condition is equivalent to
\[\text{min}\{v_p(p^d(a' - b')), d\} = \text{min}\{d, d\}\]which is equivalent to $d = d$, true.

If $c < d$, the condition is equivalent to
\[\text{min}\{v_p(p^c(a' - b' \cdot p^{d-c})), c\} = \text{min}\{c, d\}\]which is equivalent to $c = c$, true.

Now, the given condition in the problem rewrites as
\[\text{min}\{v_p(n), v_p(m + x), v_p(n), v_p(m + y)\} = \text{min}\{v_p(n), v_p(m + x), v_p(x - y)\}\]for all primes $p$. Furthermore since $v_p(n)$ is maximized, the above is equivalent to
\[\text{min}\{v_p(m + x), v_p(m + y)\} = \text{min}\{v_p(m + x), v_p(x - y)\}\]Setting $a = m + x, b = m + y$, this is true by the claim we proved above, so we are done.
Z K Y
The post below has been deleted. Click to close.
This post has been deleted. Click here to see post.
lelouchvigeo
183 posts
#71 • 1 Y
Y by alexanderhamilton124
Solution
Z K Y
The post below has been deleted. Click to close.
This post has been deleted. Click here to see post.
cursed_tangent1434
634 posts
#72
Y by
Let $\ell = \text{lcm} (1,2,\dots,10^{100})$. We start off with the following simple observation which allows us to work on $f$ with a finite domain.

Claim : For all integers $n$, $f(n)=f(n+\ell)$.

Proof : Simply note that,
\[\gcd(f(\ell+n),f(n))=\gcd(f(\ell+n),\ell) = f(\ell+n)\]which implies that $f(\ell+n) \mid f(n)$. Similarly,
\[\gcd(f(n),f(\ell+n))=\gcd(f(n),-\ell)=f(n)\]which implies that $f(n) \mid f(\ell+n)$ which in conjunction with the prior result implies that $f(n)=f(n+\ell)$ as claimed.

Now we work with the restricted function $f:\{1,2,\dots,\ell\} \to \{1,2,\dots , 10^{100}\}$ and extend the function to all of $\mathbb{Z}$ using the above claim. Now comes the most important result.

Let $p_1<p_2<\dots < p_a$ be the set of all primes dividing $\ell$. Let $M_i$ be the positive integer such that $\nu_{p_i}(f(M_i))$ is maximal under the restricted domain.

Claim : For all positive integers $x$, and primes $p_i\mid \ell$,
\[\nu_{p_i}(f(x))=\min\{\nu_{p_i}(x-M_{_i}),\nu_{p_i}(f(M_{p_i}))\}\]
Proof : If $\nu_p(f(x))=\nu_p(f(M_i))$ by the given condition,
\[\nu_p(f(x))=\min\{\nu_p(f(x)),\nu_p(f(M_i))\} = \min\{\nu_p(f(x),\nu_p{(x-M_i)})\}\]which implies that $\nu_p((x-M_i)) \ge \nu_p(f(x))$ so the claim holds. Else, $\nu_p(f(x))<\nu_p(f(M_i))$ and in this case the given condition implies,
\[\nu_p(f(x)) = \min\{\nu_p(f(M_i)),\nu_p(f(x))\}= \min\{\nu_p(f(M_i)),\nu_p(M_i-x)\}\]clearly $\nu_p(f(M_i))=\nu_p(f(x))$ is impossible so it follows that $\nu_p(f(M_i)) > \nu_p(M_i-x)$ and hence $\nu_p(f(x))= \nu_p(M_i-x)$ which finishes the proof of the claim.

We now make the above expression symmetric in $M_i$. To do this, first note that,
\[\nu_{p_i}(M_i) = \max\{\nu_{p_i}(M_1),\nu_{p_i}(M_2), \dots , \nu_{p_i}(M_a)\} = \nu_{p_i}(\text{lcm}(M_1,M_2,\dots , M_a))\]Now, let $r_i = \max\{\nu_{p_i}(M_{p_i}-x),\nu_{p_i}(M_{p_i})\}$ across all $1 \le x \le \ell$ and $x \ne M_{p_i}$. Then, consider $T \equiv -M_{p_i} \pmod{p^{r_i+1}}$. So, for $x \ne M_{p_i}$ we have,
\[\nu_{p_i}(M_{p_i}-x) = \nu_{p_i}(x-M_{p_i}+(M_{p_i}+T)) = \nu_{p_i}(x+T)\]as $\nu_{p_i}(M_i+T) > \nu_{p_i}(x-M_{p_i})$ by nature of construction. And when $x=M_{p_i}$ we similarly have,
\[\nu_{p_i}(x-M_i+M_{p_i}+T) = \nu_{p_i}(M_i+T) > \nu_{p_i}(M_i)\]as $\nu_p(x-M_i)= \infty$. Thus, for all $1\le x \le \ell$ we have,
\[\min\{\nu_{p_i}(M_{p_i}-x),\nu_{p_i}(f(M_{p_i}) = \min\{\nu_{p_i}(x+T),\nu_{p_i}(f(M_{p_i}))\}\}\]Now, by the Chinese Remainder Theorem we can pick $T$ which satisfies the prescribed congruence for each prime $p_1,p_2,\dots , p_a$ we provides us with a positive integer $T$ for which the above relation holds for all primes $p_i$ for a fixed constant $T$. Combining our two conclusions we have,
\[\nu_{p_i}(f(x))= \min\{\nu_{p_i}(x+T),\nu_{p_i}(\text{lcm}(f(M_1),f(M_2),\dots , f(M_a)))\}\]which summing across all primes $p_1,p_2,\dots , p_a$ implies
\[f(x) = \gcd(x+T,L)\]where $T$ and $L=\text{lcm}(f(M_1),f(M_2),\dots , f(M_a))$ are fixed positive constants as desired.
Z K Y
The post below has been deleted. Click to close.
This post has been deleted. Click here to see post.
ihategeo_1969
235 posts
#73
Y by
I am so trash at maths genuinely.

Let $p_1 < \dots < p_t$ be all primes $\le 10^{100}$. Let $e_i$, $a_i$ be $\max(\nu_{p_i}(x))$ and such a $x$ respectively.

Now by $Q(a_i,x)$ (where $Q(x,y)$ is the assertion) we get \[\nu_{p_i}(f(x))=\min(e_i,\nu_{p_i}(f(x))=\min(e_i,\nu_{p_i} (x-a_i))\]Now by CRT choose $m$ such that \[-m \equiv a_i+p_i^{e_i} \pmod {p_i^{e_i+1}} \iff \nu_{p_i}(m+a_i)=e_i\]Claim: $\min(e_i,\nu_{p_i}(x-a_i))=\min(e_i,\nu_{p_i}(x+m))$.
Proof: See that $\nu_{p_i}(x-a_i)=\min(e_i,\nu_{p_i}(x+m))$ unless $e_i=\nu_{p_i}(x+m)$. In each of these cases, we will analyse what can go ``wrong" and prove it cannot happen.

$\bullet$ Let $e_i<\nu_{p_i}(x-a_i)$. If $e_i>\nu_{p_i}(x+m)$ then $e_i<\nu_{p_i}(x-a_i)=\nu_{p_i}(x+m)<e_i$, contradiction.
$\bullet$ Let $e_i>\nu_{p_i}(x-a_i)$. If $e_i<\nu_{p_i}(x+m)$ then $e_i>\nu_{p_i}(x-a_i)=e_i$, contradiction.
$\bullet$ Let $e_i=\nu_{p_i}(x-a_i)$. If $\nu_{p_i}(x+m)<e_i$ then $e_i=\nu_{p_i}(x-a_i)=\nu_{p_i}(x+m)$, contradiction.

Done. $\square$

Now just choose $n=\prod_{i=1}^t p_i^{e_i}$ And hence \[\nu_{p_i}(f(x))=\min(\nu_{p_i}(x+m),\nu_{p_i}(n)) \iff f(x)=\gcd(x+m,n)\]As required.
Z K Y
N Quick Reply
G
H
=
a